Difference between revisions of "2002 AMC 12A Problems/Problem 17"

(New page: == Problem == Several sets of prime numbers, such as <math>\{7,83,421,659\}</math> use each of the nine nonzero digits exactly once. What is the smallest possible sum such a set of primes...)
 
m (Solution)
Line 21: Line 21:
 
We can indeed create a set of primes with this sum, for example the following set works: <math>\{ 41, 67, 89, 2, 3, 5 \}</math>.
 
We can indeed create a set of primes with this sum, for example the following set works: <math>\{ 41, 67, 89, 2, 3, 5 \}</math>.
  
Thus the answer is <math>\boxed{207}</math>.
+
Thus the answer is <math>\boxed{(B)207}</math>.
  
 
== See Also ==
 
== See Also ==
  
 
{{AMC12 box|year=2002|ab=A|num-b=16|num-a=18}}
 
{{AMC12 box|year=2002|ab=A|num-b=16|num-a=18}}

Revision as of 00:13, 2 July 2013

Problem

Several sets of prime numbers, such as $\{7,83,421,659\}$ use each of the nine nonzero digits exactly once. What is the smallest possible sum such a set of primes could have?

$\text{(A) }193 \qquad \text{(B) }207 \qquad \text{(C) }225 \qquad \text{(D) }252 \qquad \text{(E) }447$

Solution

Neither of the digits $4$, $6$, and $8$ can be a units digit of a prime. Therefore the sum of the set is at least $40 + 60 + 80 + 1 + 2 + 3 + 5 + 7 + 9 = 207$.

We can indeed create a set of primes with this sum, for example the following set works: $\{ 41, 67, 89, 2, 3, 5 \}$.

Thus the answer is $\boxed{(B)207}$.

See Also

2002 AMC 12A (ProblemsAnswer KeyResources)
Preceded by
Problem 16
Followed by
Problem 18
1 2 3 4 5 6 7 8 9 10 11 12 13 14 15 16 17 18 19 20 21 22 23 24 25
All AMC 12 Problems and Solutions